Prove directly from the definition of the Ito's integral

I am trying to solve the exercises from the book Stochastic differential equations -An Introduction with applications by Bernt Oksendal and I am stuck on 1 question.

Prove directly from the definition of Ito's integral that

$$\int_0^t B_s^2 dB_s=\frac{1}{3}B_t^3-\int_0^t B_s ds$$

I have tried for a couple of hours but I simply cannot prove it . I would be grateful if somebody could help me out , even a hint would be very helpful


Note that $$\begin{align*}B_T^3 &=\sum_{j=1}^n B_{s_j}^3 - B_{s_{j-1}}^3 = \sum_{j=1}^n (B_{s_{j-1}}+(B_{s_j}-B_{s_{j-1}}))^3-B_{s_{j-1}}^3 \notag \\ &= \underbrace{\sum_{j=1}^n (B_{s_j}-B_{s_{j-1}})^3}_{=: I_1}+ 3 \underbrace{\sum_{j=1}^n B_{s_{j-1}} \cdot (B_{s_j}-B_{s_{j-1}})^2}_{=:I_2} + 3 \underbrace{\sum_{j=1}^n B_{s_{j-1}}^2 \cdot (B_{s_j}-B_{s_{j-1}})}_{=: I_3} \tag{1}\end{align*}$$

We consider the terms separately and start with $I_3$:


For \begin{equation*} g^\Pi(s,w) := \sum_{j=1}^n \underbrace{B_{s_{j-1}}^2}_{\in L^2} \cdot 1_{[s_{j-1},s_j)}(s) \end{equation*}

we have \begin{align*} \mathbb{E} \left( \int_0^T |g^\Pi(s)-B_s^2|^2 \, ds \right) &\stackrel{\text{Fub}}{=} \sum_{j=1}^n \int_{s_{j-1}}^{s_j} \mathbb{E}(|B_{s_{j-1}}^2-B_s^2|^2) \, ds \\ &\leq \sup_{|s-t| \leq |\Pi|} |B_s^2-B_t^2|^2 \sum_{j=1}^n \int_{s_{j-1}}^{s_j} 1 \ \, ds \\ &\to 0 \qquad (|\Pi| \to 0)\end{align*}

where $|\Pi| := \max (s_j-s_{j-1})$ denotes the mesh size of the partition $\Pi = \{0=s_0<\ldots<s_n=T\}$. In the last step, we have used that $[0,T] \ni t \mapsto B_t$ is uniformly continuous (almost surely). From the definition of the Itô integral, it follows that

$$g^\Pi \bullet B_T = \sum_{j=1}^n B_{s_{j-1}}^2 \cdot (B_{s_j}-B_{s_{j-1}}) \stackrel{L^2}{\to} \int_0^T B_s^2 \, dB_s $$

Hence, $$I_3 \stackrel{|\Pi| \to 0}{\to} \int_0^T B_s^2 \, dB_s. \tag{2}$$


Next we consider $I_2$. To this end, let $X_j := B_{s_{j-1}} \cdot ((B_{s_j}-B_{s_{j-1}})^2-(s_j-s_{j-1})$. Then $$\begin{align*} \left\| \left( \sum_{j=1}^n X_j \right)^2 \right\|_{L^2(\mathbb{P})}^2 &\stackrel{\ast}{=} \sum_{j=1}^n \mathbb{E}(X_j^2) = \sum_{j=1}^n \mathbb{E}((B_{s_{j-1}}^2 \cdot ((B_{s_j}-B_{s_{j-1}})^2-(s_j-s_{j-1})))^2) \\ &= \sum_{j=1}^n \underbrace{\mathbb{E}(B_{s_{j-1}}^2)}_{s_{j-1}} \cdot \underbrace{\mathbb{E}( ((B_{s_j}-B_{s_{j-1}})^2-(s_j-s_{j-1}))^2)}_{\stackrel{L1.6}{=} (s_j-s_{j-1})^2} \\ &= \sum_{j=1}^n s_{j-1} \cdot (s_j-s_{j-1})^2 \leq |\Pi| \cdot \underbrace{\sum_{j=1}^n s_{j-1} \cdot (s_j-s_{j-1})}_{\leq T^2} \\ &\to 0 \qquad (|\Pi| \to 0) \end{align*}$$ For $(\ast)$ note that \begin{align*} \mathbb{E}(X_j \cdot X_k) &= \mathbb{E}(\mathbb{E}(X_j \cdot X_k|\mathcal{F}_{k-1})) \\ &= \mathbb{E}(X_j \cdot B_{s_{k-1}} \cdot \underbrace{\mathbb{E}((B_{s_j}-B_{s_{j-1}})^2)-(s_j-s_{j-1})|\mathcal{F}_{k-1})}_{= \mathbb{E}((B_{s_j}-B_{s_{j-1}})^2-(s_j-s_{j-1}) = 0}) = 0 \end{align*} for all $j>k$ where we have used that $B_{s_j}-B_{s_{j-1}}$ is independent from $\mathcal{F}_{k-1}:=\mathcal{F}_{s_{k-1}}$. Consequently, there exists a subsequence converging almost surely to $0$. Since $$\sum_{j=1}^n B_{s_{j-1}} \cdot (s_j-s_{j-1}) \to \int_0^T B_s \, ds \quad \text{a.s.} \quad (|\Pi| \to 0)$$ we conclude $$I_2 \to \int_0^T B_s \, ds \quad \text{a.s.} \quad (|\Pi| \to 0) \tag{3}$$


It remains to determine $\lim_{|\Pi| \to 0} I_1.$ To this end, we note that $$\begin{align*} \mathbb{E} \left( \left| \sum_{j=1}^n (B_{s_j}-B_{s_{j-1}})^3 \right| \right) &\leq \sum_{j=1}^n \mathbb{E}(|B_{s_j}-B_{s_{j-1}}| \cdot |(B_{s_j}-B_{s_{j-1}})^2|) \\ &\stackrel{\text{CSI}}{\leq} \sum_{j=1}^n \underbrace{\|B_{s_j}-B_{s_{j-1}}\|_{L^2(\mathbb{P})}}_{\sqrt{s_j-s_{j-1}}} \cdot \underbrace{\|(B_{s_j}-B_{s_{j-1}})^2\|_{L^2(\mathbb{P})}}_{\sqrt{3 (s_j-s_{j-1})^2}} \\ &= \sqrt{3} \cdot \sum_{j=1}^n (s_j-s_{j-1})^{\frac{3}{2}} \leq |\Pi|^{\frac{1}{2}} \cdot \underbrace{\sum_{j=1}^n (s_j-s_{j-1})}_{T} \\ &\to 0 \quad (|\Pi| \to 0) \end{align*}$$ i.e. $I_1(\Pi) \stackrel{L^1}{\to} 0$ as $|\Pi| \to 0$. Consequently, there exists a subsequence converging to $0$ almost surely.


Adding all up, we get \begin{align*} B_T^3 &\stackrel{(1)}{=} \lim_{|\Pi| \to 0} \bigg( \sum_{j=1}^n (B_{s_j}-B_{s_{j-1}})^3 + 3 \underbrace{\sum_{j=1}^n B_{s_j} \cdot (B_{s_j}-B_{s_{j-1}})^2}_{\stackrel{(3)}{\to} \int_0^T B_s \, ds} + 3 \cdot \underbrace{\sum_{j=1}^n B_{s_{j-1}}^2 \cdot (B_{s_j}-B_{s_{j-1}})}_{\stackrel{(2)}{\to} \int_0^T B_s^2 \, dB_s} \bigg) \\ &= 0 + 3 \cdot \int_0^T B_s \, ds + 3 \cdot \int_0^T B_s^2 \, dB_s \end{align*}